Difficult simplification for Arc length integral

Click For Summary
The discussion revolves around finding the arc length of the curve defined by the equation x = 3y^{4/3} - (3/32)y^{2/3} over the interval -64 ≤ y ≤ 64. The integral for arc length is set up, but the user struggles with integrating the resulting expression. A simplification of the integrand is suggested, leading to the form 4y^{1/3} + (1/16)y^{-1/3}, although the user is unclear on how to achieve this simplification. Participants clarify the need to correctly apply the formulas for arc length and suggest factoring the expression under the radical to facilitate integration. The conversation emphasizes the importance of understanding the relationship between dx and dy in the context of the problem.
nicnicman
Messages
132
Reaction score
0

Homework Statement


Find the length of the curve
x = 3 y^{4/3}-\frac{3}{32}y^{2/3}, \quad -64\le y\le 64

Homework Equations


Integral for arc length (L):
L = \int_a^b \sqrt{1 + (\frac{dy}{dx})^{2}} dx

The Attempt at a Solution


Using symmetry of the interval and the above integral for arc length I got
L = 2\int_0^{64} \sqrt{1 + (4y^{\frac{1}{3}-}\frac{1}{16y^{\frac{1}{3}}})^{2}} dy

Unfortunately, I'm having trouble integrating this beast. Using Wolfram Alpha as a last resort, I found the integrand can be simplified to this:
4y^{\frac{1}{3}}+\frac{1}{16y^{\frac{1}{3}}}
However, I can't see how Worlfram made this simplification. (A step-by-step solution was not available.)

Could someone show me how the the original integrand could be simplified to this form? After that I should be fine.
 
Last edited:
Physics news on Phys.org
The expression 4y1/3 - (1/16)y-1/3 isn't dy/dx, it's dx/dy. So the differential in your expression for L should be dy rather than dx. But, OK, to answer your actual question:

The expression under the radical sign can be simplified if you square the parenthesized expression by the rules of high-school algebra. It comes out to 16y2/3 - 1/2 + (1/256)y-2/3, so when you add 1 to it, the -1/2 becomes a +1/2, and the square root is just the same as what was formerly in the parentheses...except now the minus is a plus.
 
Thanks for the reply.

Yeah I know the dx and dy are backwards...it's just the generic formula we were given. Sorry for any confusion.

I did expand the expression under the radical and came up with the same expansion you did, but I still could not see a good way to integrate it.

That's when I turned to Wolfram and found the simplification shown in the original post. However, I don't see how to get to that simplification.
 
Last edited:
nicnicman said:
Thanks for the reply.

Yeah I know the dx and dy are backwards...it's just the generic formula we were given. Sorry for any confusion.

I did expand the expression under the radical and came up with the same expansion you did, but I still could not see a good way to integrate it.

That's when I turned to Wolfram and found the simplification shown in the original post. However, I don't see how to get to that simplification.

Just follow az-lender's lead and factor 16y^(2/3) + 1/2 + (1/256)y^(-2/3) into a perfect square.
 
nicnicman said:
Thanks for the reply.

Yeah I know the dx and dy are backwards...it's just the generic formula we were given.
It's likely that you were given three formulas: one where y is a function of x, another where x is a function of y, and a third where x and y are functions of a parameter such as t. It's important to know which one to apply to a given situation - you can't just mix and match things.
nicnicman said:
Sorry for any confusion.

I did expand the expression under the radical and came up with the same expansion you did, but I still could not see a good way to integrate it.

That's when I turned to Wolfram and found the simplification shown in the original post. However, I don't see how to get to that simplification.
 
Okay thanks for the help everyone
 
Question: A clock's minute hand has length 4 and its hour hand has length 3. What is the distance between the tips at the moment when it is increasing most rapidly?(Putnam Exam Question) Answer: Making assumption that both the hands moves at constant angular velocities, the answer is ## \sqrt{7} .## But don't you think this assumption is somewhat doubtful and wrong?

Similar threads

  • · Replies 2 ·
Replies
2
Views
2K
Replies
4
Views
2K
Replies
2
Views
2K
Replies
7
Views
2K
  • · Replies 1 ·
Replies
1
Views
1K
Replies
4
Views
2K
  • · Replies 105 ·
4
Replies
105
Views
6K
Replies
20
Views
2K
  • · Replies 6 ·
Replies
6
Views
2K
Replies
3
Views
2K